User avatar
 
LSAT-Chang
Thanks Received: 38
Atticus Finch
Atticus Finch
 
Posts: 479
Joined: June 03rd, 2011
 
 
trophy
Most Thankful
trophy
First Responder
 

Q13 - Essayist: one of the claims

by LSAT-Chang Tue Aug 16, 2011 10:36 pm

Could someone help me with this problem? I picked (A) but I've realized that the conclusion just says that laissez-faire economics is not entirely correct, so it leaves the space for some of it to be possibly correct, so (A) is wrong since it conludes about "all" the doctrines -- don't have to assume this.

But with (B)...

So the author is concluding that laissez-faire economics is not entirely correct. Why? Because one of the claims is that increasing minimum wage reduces the total number of minimum-wage jobs available. But in one study, fast-food restaurants increased minimum-wage but kept the same number of minimum-wage employees. So I thought the author was assuming something along the lines of "if one study goes against a claim of a theory, then it is not entirely correct". And in this case, since the fast food case study goes against the claim, it isn't entirely correct. But I don't get what (B) has to do with the conclusion...
User avatar
 
noah
Thanks Received: 1192
Atticus Finch
Atticus Finch
 
Posts: 1541
Joined: February 11th, 2009
 
This post thanked 2 times.
 
 

Re: Q13 - Essayist: one of the claims

by noah Fri Aug 19, 2011 4:22 pm

For this necessary assumption question, the core is as follows:

fast-food restaurants didn't see a reduction in min. wage jobs when min. wage was raised + l.f. economics claims that when you raise min. wage, you will see a reduction in total # of min. wage jobs --> l.f. economics not totally accurate.

The gap, which is hard to see, is that l.f. economics is talking about the number of min. wage jobs in total available, while the example just focuses on restaurant jobs. Perhaps the l.f. prediction is true - and when min. wage was raised there were fewer jobs available, but not in the restaurant industry. Those jobs stuck around, but there was a reduction in the number of jobs in cleaning office buildings.

(B) addresses that gap - now we can say that the # of min. wage jobs in restaurants tells us what's going on with the # of min. wage jobs in general.

(A) is wrong because we're not sure the l.f. prediction was wrong (for the reason outlined above), ad we don't need ALL doctrines of l.f. economics to be inaccurate - the conclusion is much more limited.

(C) is tempting but too strong. Do we need that there hasn't ever been even one business that decreased the number of min. wage employees after a wage increase? Of course not. It still could be true that the overwhelming majority of businesses don't do this.

(D) is strange. Apparently, the restaurants in the study did not pay minimum wage anyway! This doesn't help us - it just explains what happened. If anything, it shows we shouldn't include that data.

(E) is tempting as it does bring in wider data about jobs, but it doesn't tell us whether the restaurant data can allow us to draw the conclusion about l.f. Plus, even if the unemployment rate did increase, that doesn't tell us if the number of min. wage jobs decreased. Perhaps all the white collar workers were laid off.
 
icebreaker
Thanks Received: 0
Forum Guests
 
Posts: 8
Joined: October 20th, 2013
 
 
 

Re: Q13 - Essayist: one of the claims

by icebreaker Wed Jan 01, 2014 3:46 pm

noah Wrote:(C) is tempting - but if anything, it's the reverse of what we might want. We want studies showing that business DID decrease the number of jobs...


Hi there,

Would anyone be able to elaborate a bit more on answer choice (C)? I am still slightly confused about it.

My prephrase for this question was: the author must be assuming that another industry did not decrease their number of employees / jobs in an amount that would bring the total number of jobs down despite the fast-food industry remaining constant with their number.

I can see why B is correct but I am having trouble eliminating C.

If I negate (C) - 'studies HAVE shown that a business has decreased the number of its min-wage employees after an increase in the minimum wage' then wouldn't the author's conclusion be incorrect? (Am I wrong to equate "employees" in this answer choice to # of jobs available?)

Thank you in advance
User avatar
 
noah
Thanks Received: 1192
Atticus Finch
Atticus Finch
 
Posts: 1541
Joined: February 11th, 2009
 
 
 

Re: Q13 - Essayist: one of the claims

by noah Mon Jan 06, 2014 2:46 pm

icebreaker Wrote:
noah Wrote:(C) is tempting - but if anything, it's the reverse of what we might want. We want studies showing that business DID decrease the number of jobs...


Hi there,

Would anyone be able to elaborate a bit more on answer choice (C)? I am still slightly confused about it.

My prephrase for this question was: the author must be assuming that another industry did not decrease their number of employees / jobs in an amount that would bring the total number of jobs down despite the fast-food industry remaining constant with their number.

I can see why B is correct but I am having trouble eliminating C.

If I negate (C) - 'studies HAVE shown that a business has decreased the number of its min-wage employees after an increase in the minimum wage' then wouldn't the author's conclusion be incorrect? (Am I wrong to equate "employees" in this answer choice to # of jobs available?)

Thank you in advance


Thanks for the question. Looking over my earlier explanation, I realize I was turned around. I corrected it above.

See if that clears it up.
User avatar
 
Mab6q
Thanks Received: 31
Atticus Finch
Atticus Finch
 
Posts: 290
Joined: June 30th, 2013
 
 
 

Re: Q13 - Essayist: one of the claims

by Mab6q Wed Oct 29, 2014 6:53 pm

Noah I chose E here but I see now why it is incorrect. As you said, we don't know that the minimum wage jobs increased. However, I would argue that B is just as bad.

B says: minimum wage job availability at fast foods included in the study was representative of minimum wage jobs availability in general. Although this might be sufficient, it is not necessary because we don't need it to be representative. Maybe there is even more availability in general, more than that in the restaurants. In that case, it would not be representative (negation is supposed to kill the argument), but the argument would still follow. In fact, it could be sufficient since it would show the increase led to more minimum wage job availability, proving the conclusion.

I hope that made sense. We simply don't need the availability to be representative, it can be greater than or equal to that for the fast food restaurants.
"Just keep swimming"
User avatar
 
ohthatpatrick
Thanks Received: 3806
Atticus Finch
Atticus Finch
 
Posts: 4661
Joined: April 01st, 2011
 
This post thanked 1 time.
 
 

Re: Q13 - Essayist: one of the claims

by ohthatpatrick Sun Nov 02, 2014 1:25 am

You're hitting on an issue with Necessary Assumption that bugged me for awhile:

Sometimes the correct answer, when negated, weakens but definitely doesn't disprove.

This is disconcerting because for so many correct NA answers, the negation actually seems to be such a powerful weakener that we feel like we COULDN'T any longer draw the conclusion.

But that's not the standard by which we want to measure the Negation Test.

For instance:
All boys like sports.
Thus, Sam likes sports.

What does the author have to assume?

If you're (correctly) thinking "Sam is a boy", great!

If we negate "Sam is a boy" and get "Sam is a girl", does that falsify the conclusion?

Is it still possible that Sam, a girl, likes sports?

Sure!

Necessary Assumptions aren't commenting on the truth value of the conclusion, but rather the reasoning that got us there.

If someone were saying:
All boys like sports, therefore Sam likes sports
and we said
"Um, Sam is a girl."

This WEAKENS but does not GUARANTEE THE FALSITY of the conclusion.

We teachers complicate this confusion by using dramatic phrases like "The correct answer, when negated, should CRUSH the argument! The argument should crumble like a tower of Jenga!"

Crushing an "argument", though, is not the same as contradicting a conclusion.

In Q13, the author is making this move:
This restaurant didn't lose minimum wage jobs.
Thus, the total number of minimum wage jobs available did not decrease.

To make that move of extrapolating from one PART to the WHOLE, you DO have to assume that the PART is representative of the WHOLE.

Here, we're saying THIS RESTAURANT "didn't lose jobs", so OVERALL "didn't lose jobs". (that's compatible with staying the same number of jobs OR having an increasing number)

I realize that the argument does say THIS RESTAURANT "roughly the same", but I'm thinking of that as "didn't lose jobs" because the force of this evidence is directed at proving laissez-faire wrong ... we're trying to show that "increasing minimum wage DOES NOT reduce available jobs".

The moral of the story is that the negation doesn't have to be perfect. If you see the negation as a CLEAR weakening idea, then that answer is almost certainly correct.

Saying "the fast-food industry was NOT representative of the overall" has the same effect as "Sam is not a boy" ... it's saying "where do you get off using THAT premise as a stepping stone to THAT conclusion?"

Hope this helps.